Grenzwert einer Reihe.

Aufrufe: 790     Aktiv: 02.04.2019 um 19:00

0

Wie komm ich bei folgenden Reihen auf den Grenzwert?

 

 \(sum_{n=1}^{\infty} \frac {1} {n}\)

 \(sum_{n=1}^{\infty} \frac {1} {n^2}\)

 

Daniel hatte mal in einem Video gesagt, dass bei der ersten Reihe q=2 ist. Jedoch war das kein Video wo er dies berechnen hat. Weiß nicht wie er drauf gekommen ist.

Diese Frage melden
gefragt

Punkte: 10

 
Kommentar schreiben
1 Antwort
0

Hallo,

\(\displaystyle\sum\limits_{n=1}^\infty \dfrac{1}{n}\) stellt die harmonische Reihe dar.

Allgemein konvergiert \(\displaystyle\sum\limits_{n=1}^\infty \dfrac{1}{n^\lambda}\) nur für \(\lambda > 1\).

Der Beweis, dass \(\displaystyle\sum\limits_{n=1}^\infty \dfrac{1}{n^2}=\lim\limits_{n\to\infty}H_n^{(2)}=\dfrac{\pi ^2}{6}\) gilt, findest du vermehrt im Internet, z.B. hier.

Diese Antwort melden
geantwortet

Sonstiger Berufsstatus, Punkte: 16.5K

 

Kommentar schreiben